Help me pleasee anyone

Answers

Answer 1
x = -4 is the correct answer

Related Questions

What conclusion can be derived by comparing the central tendencies of the two data sets?
A: (7, 6, 3, 1, 6, 2, 4, 6, 3, 5;
B: {2, 2, 2, 3, 4, 5, 2, 8, 7, 6}

Answers

A is the correct answer because u should know

Choose ALL of the ordered pairs that are solutions to the equation.
5y = 2x - 7
a) (6, 0)
b) ( 7, 21)
c) (8, 18)
d) ( -3, 4)

Answers

b) ( 7, 21)

Step-by-step explanation:

5(7)=2(21)-7

35=42-7

35=35

Answer:

None of them

Step-by-step explanation:

a: 5*0=2*6-7 False

b: 5*21=2*7-7 False

c: 5*18=2*8-7 False

d: 5*4=2*-3-7 False

Which function of x has the least value for they intercept?

Answers

The answer is 3x^2-6x+13

Solve the system of equations.
6x – Зу = 3
-2x + y = 14
1.What number would you multiply the second equation
by in order to eliminate the x-terms when adding to the
first equation?
2.What number would you multiply the first equation by in
order to eliminate the y-terms when adding to the
second equation?

Answers

Answer:

1. Multiply by 3

2. Multiply by 1/3

Step-by-step explanation:

6x - 3y = 3

-2x + y = -14

1. In order to eliminate the x-terms, they would both have to equal the same number with opposite signs (positive and negative). Since we are only touching the second equation, how can we get from 2 to 6? We can multiply 2 by 3 to get to 6 (negative 6 in the end.) This would result as follows:

6x - 3y = 3

-6x + 3y = 42

2. In order to eliminate the y-terns through multiplying with the first equation, we have to figure out how to make 3y into 1y. We can do this by adding its reciprocal, 1/3. 1/3 × 3 = 1. This would result as follows:

2x - y = 1

Explain how you can find the constant of proportionality from a graph representing a proportional relationship when it shows a point with an x - value of 1 and if it doesn't show an x - value of 1 .

Answers

tienees que fijartelos extrreemos

Which rule is a recursive rule for the sequence 1, -6, 36, -216,...?

( I'll put the answers a screenshot )

PLZZ Help I've been stuck for so long

Answers

Answer:

The last answer

Step-by-step explanation:

We can check

For example,

a5=-6×a4

a5=-6×(-216)

a5=1296

SOMEONE PLEASE HELP ME ASAP !!!

Mrs. Pregnals's biology classes are going on a field trip. She has them break into groups of three or four students in order to do labs. She has a total of 120 students and 32 total groups are formed.

If t represents the number of three person groups and frepresents the number of four person groups, then write a system of equations that models this situation.

How many four person groups are there? Only an algebraic solution will receive full credit.

PLEASE HELP !!!!

Answers

x + y = 32 (1)

3x + 4y = 120 (2)

x = 8; y = 24

Step-by-step explanation:

Total students = 120

Number of groups = 32

If the number of three person groups = x

Number of four person groups = y

write a system of equations that models this situation

x + y = 32 (1)

3x + 4y = 120 (2)

From (1)

x = 32 - y

Substitute x = 32 - y into (2)

3(32 - y) + 4y = 120

96 - 3y + 4y = 120

- 3y + 4y = 120 - 96

y = 24

Substitute y = 24 into (1)

x + y = 32

x + 24 = 32

x = 32 - 24

x = 8

x = 8; y = 24

pls help and pls provide workings ​

Answers

(x-h)²+(y-k)²=r²

(3-h)²+(3-0)²=5²

h²-6h+9+9=25

h²-6h-7=0 —> (h-7)(h+1)=0

h=7 , h=-1

h=7 —> (x-h)²+y²=r² —> (x-7)²+y²=25 —>

x²-14x+49+y²=25 —> x²+y²-14x+24=0

h=–1 —> (x-h)²+y²=r² —> (x+1)²+y²=25 —>

x²+2x+1+y²=25 —> x²+y²+2x–24=0

I hope it helped you ^_^

Determine which quadratic equations have
2 real solutions. Select two that apply.

Answers

Answer:

A & D

First and last

Step-by-step explanation:

Which are the solutions of x2 = -7x – 8?
Ž
17 7
4
+
17
4
-

그를
+
ZIA
+
V 17
4
4
7 - 17 7 + /17
-
-7 - V 17 -7 + v17 .
2

Answers

Answer:

( - 7 + √17 ) / 2, ( - 7 - √17 ) / 2

Step-by-step explanation:

x^2 = - 7x - 8

x^2 + 7x + 8 = 0

Here,

a = 1

b = 7

c = 8

D = b^2 - 4ac

= 7^2 - 4 ( 1 ) ( 8 )

= 49 - 32

D = 17

x = - b ± √D / 2a

= - 7 ± √17 / 2 ( 1 )

x = ( - 7 + √17 ) / 2, ( - 7 - √17 ) / 2

Titus works at a hotel. Part of his job is to keep the complimentary pitcher of water at least half full and always with ice. When he starts his shift, the water level shows 8 gallons, or 128 cups of water. As the shift progresses, he records the level of the water every 10 minutes. After 2 hours, he uses a regression calculator to compute an equation for the decrease in water. His equation is W –0.414t + 129.549, where t is the number of minutes and W is the level of water. According to the equation, after about how many minutes would the water level be less than or equal to 64 cups?

Answers

Answer:

Step-by-step explanation:

If we are looking for how long it will take the level to be less than or equal to 64 cups, we are solving an inequality as opposed to an equation. It would look like this:

-.414t + 129.549 ≤ 64  We subtract 129.549 from both sides to get

-.414t ≤ -65.549 and then divide by -.414. But because we are dividing by a negative number we need to change the way the inequality is pointing:

t ≥ 158.3 minutes which is about 2.6 hours

Answer:

t ≥ 158.33, or rounded, t ≥ 160.

Explanation:

The equation we're given is

W = -0.414t + 129.549, which an be written as

-0.414t + 129.549 = W.

We are asked to find the amount of time it would take for the water level to be less than or equal to 64 cups.  Plugging this information in, we have:

-0.414t + 129.549 ≤ 64

To solve this, we first cancel 129.549 by subtracting from both sides:

-0.414t + 129.549 - 129.549 ≤ 64 - 129.549

-0.414t ≤ -65.549

Now divide both sides by -0.414:

-0.414t/-0.414 ≤ -65.549/-0.414

t ≥ 158.33

(When we multiply or divide both sides of an inequality by a negative number, we must flip the inequality symbol)

Kasey has three pieces of wood that measure 9 inches, 12 inches, and 21 inches. Can she make a right triangle
with the three pieces of wood (without cutting or overlapping)?

Answers

Answer: No

Step-by-step explanation:

Add 2 sides together. If the sum of greater than the length of the 3rd side for all 3 options it can form a right triangle if it doesn’t than it can’t:

9 + 12 = 21 which is not greater than the 3rd side length of 21 so this cannot form a right triangle.

The answer would be (no)

The cars of a circular Ferris wheel at an amusement park are equally spaced about the wheel's circumference. They are numbered consecutively beginning with 1. The cars numbered 14 and 30 lie on opposite ends of a diameter. How many cars are on the Ferris wheel?

Answers

Answer:

32 cars

Step-by-step explanation:

we know that 14 and 30 lie on opposite sides of the ferris wheel, so we need to calculate how many cars are inbetween car number 14 and carn number 30. since each car is numbered starting from 1, we know that the cars do not skip any numbers. after counting, we know that there are 16 cars inbetween car number 14 and car number 30. since there are 2 sides of a circle we have to double our number by 2. so 16×2 is 32

Math step-by-step:

30-14=16

16×2=32

Ryan rented a truck for one day. There was a base fee of $16.99, and there was an additional charge of 74 cents for each mile driven. Ryan had to pay $133.17 when he returned the truck. For how many miles did he drive the truck?

Answers

Answer:

157 miles

Step-by-step explanation:

total cost = flat fee + cost per mile * miles

133.17 = 16.99 + .74 * m

Subtract 16.99 from each side

116.18 = .74m

Divide each side by .74

116.18 / .74 = .74m/.74

157 = m

I NEEDDDD D HELPPPPP!!!

Answers

Answer:

70 degrees

Step-by-step explanation:

there are multiple ways, as all the trigonometric functions are related.

let's try it this way :

consider the missing side to be the radius of a circle.

then 41 would be the cos value of the angle (multiplied by that radius, of course).

so, let's calculate the missing side per Pythagoras

c² = a² + b² = 113² + 41² = 12769 + 1681 = 14450

c = 120.21

and now

41 = cos(?) × 120.21

cos(?) = 41/120.21 = 0.341...

? = 70.0576... ≈ 70 degrees

Answer:

70°

Step-by-step explanation:

tangent θ = (opposite side / adjacent side)

tangent θ = (113 / 41)

θ = arctan (113 / 41)

θ = 70.0576°

*Note: "arctan" is the same as "inverse tangent"

**Note: "soh-cah-toa" (and "cho-sha-cao" for sec, csc, & cot) is an easy mnemonic device to remember the trig functions (sin, cos, & tan) and their relations to the sides of a given angle.

Which represents the inverse of the function f(x) = 4x?
h(x) = x + 4
h(x) = x-4
h(x) = 3/4x
h(x) = 1/4x

Answers

Answer:

Let the inverse of f(x) be h(x):

[tex]{ \tt{h(x) = \frac{1}{4x} }}[/tex]

Which is the correct form of the partial fraction decomposition for the expression StartFraction 4 x cubed + 3 x squared Over (x + 1) squared (x squared + 7) squared EndFraction?

Answers

Given:

The expression is:

[tex]\dfrac{4x^3+3x^2}{(x+1)^2(x^2+7)}[/tex]

To find:

The correct form of the partial fraction decomposition for the given expression.

Solution:

We have,

[tex]\dfrac{4x^3+3x^2}{(x+1)^2(x^2+7)}[/tex]

By partial fraction decomposition, it can be written as:

[tex]\dfrac{4x^3+3x^2}{(x+1)^2(x^2+7)}=\dfrac{A}{x+1}+\dfrac{B}{(x+1)^2}+\dfrac{Cx+D}{x^2+7}[/tex]           ...(i)

[tex]\dfrac{4x^3+3x^2}{(x+1)^2(x^2+7)}=\dfrac{(x+1)^2(Cx+D)+(x+1)(x^2+7)A+(x^2+7)B}{(x+1)^2(x^2+7)}[/tex]

[tex]4x^3+3x^2=(x+1)^2(Cx+D)+(x+1)(x^2+7)A+(x^2+7)B[/tex]

[tex]4x^3+3x^2=x^3A+x^3C+x^2A+x^2B+2x^2C+x^2D+7xA+xC+2xD+7A+7B+D[/tex]

[tex]4x^3+3x^2=x^3(A+C)+x^2(A+B+2C+D)+x(7A+C+2D)+7A+7B+D[/tex]

On comparing both sides, we get

[tex]A+C=4[/tex]

[tex]A+B+2C+D=3[/tex]

[tex]7A+C+2D=0[/tex]

[tex]7A+7B+D=0[/tex]

On solving these equations, we get

[tex]A=\dfrac{23}{32},B=-\dfrac{1}{8},C=\dfrac{105}{32},D=-\dfrac{133}{32}[/tex]

Substituting these values in (i), we get

[tex]\dfrac{4x^3+3x^2}{(x+1)^2(x^2+7)}=\dfrac{\dfrac{23}{32}}{x+1}+\dfrac{-\dfrac{1}{8}}{(x+1)^2}+\dfrac{\dfrac{105}{32}x-\dfrac{133}{32}}{x^2+7}[/tex]

Therefore, the required partial fraction decomposition is:

[tex]\dfrac{4x^3+3x^2}{(x+1)^2(x^2+7)}=\dfrac{\dfrac{23}{32}}{x+1}+\dfrac{-\dfrac{1}{8}}{(x+1)^2}+\dfrac{\dfrac{105}{32}x-\dfrac{133}{32}}{x^2+7}[/tex]

Answer:

A on Edg

Step-by-step explanation:

got it right

On an electricity bill of $40.80, Mrs.
Miller pays only $38.76 because she pays
before a certain date. What percent
discount is Mrs. Miller allowed?

Answers

Answer:

100%_$40.80

? × _$38.76

Step-by-step explanation:

100%×$38.76÷$40.80=95%

Answer:

40.80

Step-by-step explanation:

Please help me with this

Answers

Answer:

A

Step-by-step explanation:

first answer gets marked brainliest!!!

Answers

Answer:

in neighbourhood q the total number of people in the nine families I higher than that in neighbourhood s because of we add up the number of people in the families in neighbourhood q we get a total of 35 yet we have a total of only 27 people in the nine families in neighbourhood s

hope that helps ❤

Travis makes and sells fishing lures. He
has 56 fishing lures that he is putting into
boxes. Each box can hold 5 fishing lures.
How many boxes can Travis fill
completely?

Answers

11 boxes.

Explanation:
You divide 56 by 5 which will give you 11.2. Round down and you will get 11.

James is 8 years old. Last year, his father was 4 times as old as he was. In how many years' time will their combined age be 51?

Answers

Last year He was 7 years old.

His father is 4 times the age 7 x 4 = 28

Last year his dad was 28, so this year he is 29.

Combined age this year = 29 + 8 = 37

51 - 37 = 14 years

14/ 2 people = 7 years

8 + 7 = 15

29 + 7 = 36

15 + 36 = 51

It will take 7 years.

Solve the inequality 4 is greater than n over -4

Answers

Answer:

4 > [tex]-\frac{n}{4}[/tex]

-16 > n

Step-by-step explanation:

Greater than would mean that the "arrow" is not facing the number. It would mean it is larger than the other number. Also, you have to construct the other part of the inequality which would be [tex]-\frac{n}{4}[/tex].

4×4 > [tex]-\frac{n}{4}[/tex]×4

16 > -n

-16 > n

How to find the area of this figure pls?

Answers

6 x 10 should be the answer

Kara Danvers (Supergirl) has always relied on her strength to win fights. But what happens when she meets an alien just as strong? Her sister is training her to be a more technical fighter so that Supergirl can meet any challenge. The data below record the significant strikes during randomly selected training sessions 6 months apart. Is Kara showing improvement in her fighting?

Strikes (pre): 29 32 44 34 19

Strikes (post): 51 45 68 92 64


Write the Hypotheses:

H_0:_______________, the mean difference in ___________ between ____________ and ______________ is _______________

H_a:_______________, the mean difference in ___________ between ____________ and ______________ is _______________

Answers

Answer:

H0 : The mean difference in strikes between pre and post strike is equal to 0

H1 : The mean difference in strikes between pre and post strike is not equal to 0

Step-by-step explanation:

The scenario described abibe meets the condition for a paired (dependent) sample t test as the same subject is involved in both the pre and post strike data. Therefore measurement from the strike(pre) must be paired with measurement from the strike(post) data.

For a paired test involving a single subject ; The mean difference in the two sample is 0 ; that is both are the same while the alternative negates the null by claiming that the mean difference is not equal to zero.

If $10,500 is borrowed for 5 years at an annual simple interest rate of 1.73%, how much interest will be paid if the entire loan is paid off at the end of the fifth year? Enter the answer in dollars and cents, and round to the nearest cent, if needed. Do not include the dollar sign. For example, if the answer is $0.61, only the number 0.61 should be entered.

Answers

Answer:

Step-by-step explanation:

So, yearly $2100 is paid, plus the 1.73% interest, so 1.73% interest is $2136.33every year interest stacks on itself so youll take 1.73% of the year 1 total with interest.  

Year 1 total: $2136.33

Year 2 total:$2173.29

Year 3 total:$2210.89

Year 4 total:$2249.14

Year 5 total:$2288.05

Total Paid After 5 years:$11057.70

Year 1 total interest:$36.33

Year 2 total interest:$36.96Year 3 total interest:$37.60Year 4 total interest:$38.25Year 5 total interest:$38.91

Total Interest paid after 5 years:$188.05

PLS how to do this 3 sums

Answers

Answer:

Step-by-step explanation:

7) PQ // RS,  RQ is transversal

x = 38 {Alternate interior angles are congruent}

In ΔPQR ,

x + y + ∠P  = 180   {Angle sum property of triangle}

38 + 90 + y = 180

  128 + y =  180

           y = 180 - 128

y = 52

9) Construct XY // AB

XY // AB  & AC is transversal

∠1 = ∠4     ---------(I)      {alternate interior angles are equal}

XY // AB  & BC is transversal

∠3 = ∠5     ---------(II)      {alternate interior angles are equal}

∠1 + ∠2 + ∠3 = 180   {straight line angles}

∠4 + ∠2 + ∠5 = 180    {From (I) & (II) }

Sum of three angles of triangle is 180.Hence proved.

Point P is the incenter of DEF. Point P is the point of concurrency of the angle bisector. Find PV

Answers

Answer:

12

Step-by-step explanation:

because line FB divides triangle f v w into two equal halves therefore line WP is equal to line PV which is equal to 12

The required value of the PV is 12 in triangle DEF where p is the incenter.


Point P is the incenter of ΔDEF. Point P is the point of concurrency of the angle bisector. PV to be determined.

What is incenter?

The incenter of a triangle is the intersection point of the internal angle bisectors of the triangle. In additional expressions, it can be clarified as the point where the internal angle bisectors of the triangle and perpendicular distance from this point to any of the sides of triangle is the same.

Since in triangle FGH, P is the incenter and PW, PV,PX is perpendicular distance to the sides HF, GF and GH respectively.
From image,
PW = 12
So by the property of incenter, perpendicular distance from this point to any of the sides of triangle is the same.

PW = PV
PV = 12


Thus, the required value of the PV is 12 in triangle DEF where p is the incenter.

Learn more about incenter here:
https://brainly.com/question/27868922
#SPJ2

You just decided to open a savings account and went to the bank and made an initial deposit. You decide to save another set amount each month. Without any interest being paid, the equation =20+50





represents the amount of money,
y
, in your savings account after
m
months. Select what the slope and y-intercept represent in this problem.

(Choose 2)

Answers

Answer:

Options (1) and (4)

Step-by-step explanation:

Given equation for the deposit in the saving account is,

y = 20x + 50

Here, y = Amount of money in the account

20 = money added in the account every month

50 = Initial deposit

By comparing this equation with the slope-intercept form of the equation of a line,

y = mx + b

Here, slope m = money added in the account every month

And b = y-intercept

Therefore, Options (1) and (4) will be the answer.

What is the answer I need help ASAP be truthful

Answers

Answer:

your closest number should be A

Answer:

A.√99

Step-by-step explanation:

10^2 = 100

Or vise versa √100 = 10

The closest number to 100 is 99

So √99 is closest to 10

Other Questions
Anybody knows the answers to the monomial functions ? Answer two questions about Equations A and B:A. 3(x+2) = 18B.r + 2 = 61) How can we get Equation B from Equation A? Do 8 plz find OS thanks A kite is a quadrilateral with two pairs of adjacent, congruent sides. The vertex angles are those angles in between the pairs of congruent sides. Prove the diagonal connecting these vertex angles is perpendicular to the diagonal connecting the non-vertex angles. Be sure to create and name the appropriate geometric figures. This figure does not need to be submitted. a family has two children. what is the probability that both are girls, given that at least one is a boy Describe how to transform the graph of f(x) = x2 to obtain the graph of the related function g(x).Then draw the graph of g(x). 1. g(x) = f(x + 1)2. g(x) = f(x) - 2Please help i also need to graph What value of x will make the equation true?(Square Root of 5) (square root of 5)=x Who was Nicolas Copernicus? What is the change in enthalpy of the first reaction below, given the enthalpies of the other two reactions?Here's the reactions: There was a formatting issue with the specific chemistry symbols, there all correct in the picture below 2C(s) + O2(g) 2CO(g) C(s) + O2(g) CO2(g) H0= -394 KJ/molCO(s) + 1/2 O2(g) CO2(g) H0= -283 KJ/molThere was a formatting issue with the specific chemistry symbols, there all correct in the picture below Why did the US government choose to imprison Japanese Americans? Study the following two class interfaces: class Question { public: Question(); void set_text(string new_text); void set_answer(string new_answer); void display() const; private: string text; string answer; }; class ChoiceQuestion : public Question { public: ChoiceQuestion(); void set_text(string new_text); }; Which member function from the Question class is overridden in the ChoiceQuestion class? they could not decide which one they like and ....... the and they didn't brother During the Great Migration period of 1830 to 1890 German Jews left Germany.. a. because of a wave of anti-Semitism that swept through central Europc.b. because of quotas on marriage licenses to Jews was enforced in an attempt to reduce the Jewish population. c. as curfews, special taxes and housing segregation in Germany pushed Jews out. d. All of the other responses are true. Find the value of x in each case: Acute is to chronic as temporary is to(persistent,sick,pretty,or narrow) This table shows the relationship of the total number of pieces of fruit to the number of bananas.Why is StartFraction 6 Over 5 EndFraction not equivalent to Three-halves? Is it likely an extremist party to emerge on either end of the political spectrum? Find sin 0 A. 16/20B. 12/16C. 12/20D. 16/12 convert into power notation -1/81 Identify the equation of the circle that has its center at (9, 12) and passes through the origin.